Blueprint LSAT Prep's ongoing ask-an-instructor extravaganza Forum

Special forum where professionals are encouraged to help law school applicants, students, and graduates.
Post Reply
User avatar
arcanecircle

New
Posts: 87
Joined: Sun Sep 16, 2012 12:33 am

Re: bp shinners’ semi-weekly office hours

Post by arcanecircle » Tue Nov 13, 2012 5:26 pm

Thanks! 8)

ruhl88

New
Posts: 27
Joined: Thu Sep 22, 2011 11:11 am

Re: bp shinners’ semi-weekly office hours

Post by ruhl88 » Tue Nov 13, 2012 5:27 pm

Hey, Shinners!

Can you please explain how you would attack the question in PT 57 S2 Q25?

After thinking about the argument and ACs, I narrowed it down to (C) and (E). My gut told me (E) was too strong, yet I still chose it.

Much appreciated.

User avatar
bp shinners

Gold
Posts: 3086
Joined: Wed Mar 16, 2011 7:05 pm

Re: bp shinners’ semi-weekly office hours

Post by bp shinners » Tue Nov 13, 2012 5:45 pm

ruhl88 wrote: After thinking about the argument and ACs, I narrowed it down to (C) and (E). My gut told me (E) was too strong, yet I still chose it.
This one's a little mean.

So I have a Must Be True question. I can definitely diagram it. But before I do that, a quick note.

This stimulus tells me that "THE law of the city..." - that article means that this is the sole law of the city of Weston. It's not A law of the city of Weston, but THE law, so this is the only one they have on registration of mayoral campaigns.

Anyway, the law breaks down to:
If $100+ AND ~R(esident) AND ~F(ormer)R(esident) -> R(egistered)C(ity)C(ouncil)

That series of sufficient conditions is the only series of sufficient conditions that forces the campaign to register the contribution (for the reason stated above). So this is, really, a biconditional. Either I accepted money from a ~R/~FR in excess of $100 and have to RCC, or I didn't and I don't.

As you said, E is too strong. While I know that Brimley doesn't HAVE TO RCC, there's nothing stopping him from doing it anyway. Maybe he wants to maintain transparency in the election. Maybe he made a pledge to reveal all contributions, even if the law didn't require it. Just because he doesn't have to doesn't mean he won't.

C, on the other hand, is very weak, and aligns with what I know. C just says that Brimley didn't have to RCC. Well, I know that, because he didn't accept any contributions from ~R/~FR.

Note that if the first word in this stimulus was "A", C would not be correct, as there might have been another law that required him to RCC these contributions. This question is very mean because that single article makes all the difference.

User avatar
boblawlob

Silver
Posts: 519
Joined: Mon Oct 11, 2010 7:29 pm

Re: bp shinners’ semi-weekly office hours

Post by boblawlob » Wed Nov 14, 2012 12:47 am

I'm working on the Cambridge LSAT Weaken packet and I've run into a little quirk on the LSAT that I would like some light shed on.


PT 37-S4-Q18
Conclusion: Dinos disappeared because of a comet.
Evidence: A comet could've hit earth, caused a cloud of dust to enshroud the planet, and then cooled the planet to freeze the dinos.

I know B is the best answer, but C looked tempting because it stated that there was insufficient evidence to make the determination that dinos died from effects of the cloud dust. So one person could eliminate C because it doesn't weaken and it doesn't strengthen; it just said info isn't there...so you can't make any further assessment on the conclusion of the stim based on C.

Flash backwards

PT 37-S4-Q2

Argument: VNO found in nose. VNO in animals is completely known compared to the ones in humans; most humans have VNO that is detectable but yet super tiny. When scientists stimulate VNO cells, the people who are stimulated feel some smelling sensations.
Conclusion: VNO is a sensory organ in most humans.

A is the right answer. The other answer choices were pretty bad, but A was bad as well. A states that we dont know if researchers stimulated only VNO cells in the noses.

Why does it seem like "the absence of absolute evidence" is the answer choice for some questions and not others? Maybe I'm analyzing the answer choice for the dinos question wrong, but I'm just confused about the similarity in type of these two answer choices.

What are things to look for and avoid in Weaken answer choices, especially the high level questions?

Much thanks.

ruhl88

New
Posts: 27
Joined: Thu Sep 22, 2011 11:11 am

Re: bp shinners’ semi-weekly office hours

Post by ruhl88 » Wed Nov 14, 2012 11:32 am

bp shinners wrote:
ruhl88 wrote: After thinking about the argument and ACs, I narrowed it down to (C) and (E). My gut told me (E) was too strong, yet I still chose it.
This one's a little mean.

So I have a Must Be True question. I can definitely diagram it. But before I do that, a quick note.

This stimulus tells me that "THE law of the city..." - that article means that this is the sole law of the city of Weston. It's not A law of the city of Weston, but THE law, so this is the only one they have on registration of mayoral campaigns.

Anyway, the law breaks down to:
If $100+ AND ~R(esident) AND ~F(ormer)R(esident) -> R(egistered)C(ity)C(ouncil)

That series of sufficient conditions is the only series of sufficient conditions that forces the campaign to register the contribution (for the reason stated above). So this is, really, a biconditional. Either I accepted money from a ~R/~FR in excess of $100 and have to RCC, or I didn't and I don't.

Note that if the first word in this stimulus was "A", C would not be correct, as there might have been another law that required him to RCC these contributions. This question is very mean because that single article makes all the difference.
Not sure I follow the biconditional aspect...how exactly is this a biconditional? Also, Wouldn't all three requirements i.e $100 + ~R and ~FR have to be met in order to trigger the RCC part?

Btw, the difference between "A" law and "the " law you pointed out is brilliant. Completely missed that.

Want to continue reading?

Register now to search topics and post comments!

Absolutely FREE!


User avatar
bp shinners

Gold
Posts: 3086
Joined: Wed Mar 16, 2011 7:05 pm

Re: bp shinners’ semi-weekly office hours

Post by bp shinners » Fri Nov 16, 2012 11:14 am

ruhl88 wrote: Not sure I follow the biconditional aspect...how exactly is this a biconditional? Also, Wouldn't all three requirements i.e $100 + ~R and ~FR have to be met in order to trigger the RCC part?
It's biconditional because this tells me that it is THE ONLY law of the city of Weston. So if something is going to make you RCC, then it's this law (which entails those three requirements).

And yep, it does require all three of those things. You have to receive a $100 donation from someone who isn't a resident and never lived in Weston. If any of those aren't met, then you don't have to register. Brimley only took donations from residents (doesn't meet the second criteria) and former residents (doesn't meet the third criteria), so he didn't have to register anything.

User avatar
bp shinners

Gold
Posts: 3086
Joined: Wed Mar 16, 2011 7:05 pm

Re: bp shinners’ semi-weekly office hours

Post by bp shinners » Fri Nov 16, 2012 11:33 am

boblawlob wrote: PT 37-S4-Q18
Conclusion: Dinos disappeared because of a comet.
Be careful here - the author doesn't conclude that the dinos did die from a comet. He just says it's a plausible explanation. In other words, the author thinks it's possible, but we have no idea how likely he thinks it is.
Evidence: A comet could've hit earth, caused a cloud of dust to enshroud the planet, and then cooled the planet to freeze the dinos.
The evidence explains how it could have happened, which does make it plausible. The flaw here is that the author never establishes whether there's evidence out there that counters this belief - a very subtle flaw, but it's there. Whenever I analyze the pros of a theory without analyzing the cons, I have a bad comparison flaw.
I know B is the best answer, but C looked tempting because it stated that there was insufficient evidence to make the determination that dinos died from effects of the cloud dust. So one person could eliminate C because it doesn't weaken and it doesn't strengthen; it just said info isn't there...so you can't make any further assessment on the conclusion of the stim based on C.
My conclusion was just that the theory is plausible. B gives me specific information to call the theory into doubt, which makes it less likely to be plausible. C just tells me we can't disprove it, which still allows it to be plausible.
Flash backwards

PT 37-S4-Q2

Argument: VNO found in nose. VNO in animals is completely known compared to the ones in humans; most humans have VNO that is detectable but yet super tiny. When scientists stimulate VNO cells, the people who are stimulated feel some smelling sensations.
Conclusion: VNO is a sensory organ in most humans.
Yep. Notice the strength difference between this conclusion and the other. In the first argument, we were told the theory is plausible. Here, we are told that the VNO is a functioning sensory organ. This conclusion is much stronger than the conclusion about the dinosaurs. The author actively believes this one; the author didn't necessarily believe the comet caused the dino death, but only that it was a possibility.

The flaw here, necessarily, is that our study was bad for some reason. The only proof for the conclusion is the study, so I'm looking for something that calls the study into question.

A does that. It doesn't have to actively tell me the study was bad to weaken this; all it has to do is call the study into question.
Why does it seem like "the absence of absolute evidence" is the answer choice for some questions and not others? Maybe I'm analyzing the answer choice for the dinos question wrong, but I'm just confused about the similarity in type of these two answer choices.
There are two differences between this questions that let the same type of answer be right in one and wrong in the other.

The first is the differing strengths of the conclusion. The first says the theory is plausible; the second says something does, in fact, exist.

The second, and probably more important, is what exactly the answer choices are 'attacking'.

In the dino question, the AC is 'attacking' evidence the author didn't even cite for his argument. His argument was that there's a logical explanation for how it could work, so therefore the theory is plausible. Telling me that some other, unrelated evidence doesn't prove or disprove it still lets his explanation stand, allowing it to be plausible.

In the VNO question, however, answer choice A calls the study into question. That study is the only evidence (other than its existence) the author cites to prove his point. I don't have to tell you the study was wrong in order to weaken it - just that it could have been wrong. So since A tells me that the study that is the only thing propping up this conclusion might be wrong, it weakens the conclusion.

Subtle but important difference between the two.

As a quick note, there's a difference here between strengthen and weaken questions. For both, you're finding the flaw in the argument. For a strengthen question, I want to make that assumption more likely to be true; for a weaken, I want to make it less likely to be true (with a caveat, as described below).

For strengthen questions, you need to actively say something, because just mentioning the flaw is not going to help the argument. So I'm going to say something that allows me to actively see how the assumption (how the flaw works in the argument) is more likely to be true than before I read that statement.

For a weaken question, however, I can actively say something that makes me think the assumption is less likely to be true, but I don't have to. Simply pointing the assumption out also weakens the argument.

So in the dino game, C doesn't point the assumption out (that there might be counterevidence) - it cites the inability of new evidence to (dis)prove a point, which isn't very helpful. In the VNO one, however, it points out the assumption - that the study might be flawed. That weakens.

ruhl88

New
Posts: 27
Joined: Thu Sep 22, 2011 11:11 am

Re: bp shinners’ semi-weekly office hours

Post by ruhl88 » Fri Nov 16, 2012 6:27 pm

Shinners,

One more for you, and IMO, it's a nasty one. Pt 56 S3 Q20: Psychotherapy question.

I was initially confused by the question stem because I did not not whether it was asking for a Sufficient Assumption or Necessary Assumption. Narrowed it down to (C) and (D).

Thank you!

meandme

New
Posts: 74
Joined: Wed Feb 15, 2012 9:36 pm

Re: bp shinners’ semi-weekly office hours

Post by meandme » Sat Nov 17, 2012 5:10 pm

Hey Matt
How would I diagram this?
V cannot be in unless both H and M are in.

I thought I could just read the "unless" as "if not". This what I got

~h and ~m --> ~v
CP V --> H or M

The "both" is throwing me off.

Thanks

Want to continue reading?

Register for access!

Did I mention it was FREE ?


User avatar
bp shinners

Gold
Posts: 3086
Joined: Wed Mar 16, 2011 7:05 pm

Re: bp shinners’ semi-weekly office hours

Post by bp shinners » Sun Nov 18, 2012 1:45 pm

ruhl88 wrote:Shinners,

One more for you, and IMO, it's a nasty one. Pt 56 S3 Q20: Psychotherapy question.

I was initially confused by the question stem because I did not not whether it was asking for a Sufficient Assumption or Necessary Assumption. Narrowed it down to (C) and (D).

Thank you!
"Must be assumed" tells me necessary.

The argument breaks down like this:
Psychotherapists on radio shows are expected to entertain.
Entertaining people is almost always at odds with providing high-quality psychological help.
____________________________________________________
Psychotherapists should never provide psychotherapy on talk shows.

The jump here is between providing less-than-high-quality psychological help and not providing any help at all. Maybe providing mediocre-quality help is good enough. Maybe just raising awareness of these issues does more good than giving less-than-high-quality help does harm. I need these two ideas to be linked together for this argument to work.

That's what E does for me. It links my two ideas together.

D has 2 problems. First, it talks about 'most members'. For reasons of argument construction and how the LSAT works, an answer with 'most' is very rarely going to be a correct answer in an assumption question. In a sufficient or necessary assumption question, be very, VERY wary of answer choices that say 'most'. Second, I know that these radio psychotherapists are expected to entertain an audience. I don't care if that's because it's what the audience demands or what their bosses demand. I just care that it's what's expected of them. D is just telling me something I already knew - that they must entertain.

C is close, but it's overkill for two reasons. First, it says that psychotherapy should never be provided in this context. Well, my conclusion is just about psychotherapists giving this advice. That still leaves open the possibility for other, non-psychotherapists to give psychotherapy over the radio (like Adam Carolla). The second is that it says 'any chance'. The stimulus just talks about a high chance of being less-than-high-quality. These two disconnects mean that C is too strong.

User avatar
bp shinners

Gold
Posts: 3086
Joined: Wed Mar 16, 2011 7:05 pm

Re: bp shinners’ semi-weekly office hours

Post by bp shinners » Sun Nov 18, 2012 1:46 pm

meandme wrote:Hey Matt
How would I diagram this?
V cannot be in unless both H and M are in.

I thought I could just read the "unless" as "if not". This what I got

~h and ~m --> ~v
CP V --> H or M

The "both" is throwing me off.

Thanks
Yep, you can just change 'unless' to 'if not'. However, you only negated H and M when you did that - you also have to negate the 'and'.

So "If not both H and M" is the same as saying, "If I don't have H or M." That diagrams as:
~H or ~M -> ~V
V-> H and M

And that's what the original is saying - if I have V, then I have both H and M.

ruhl88

New
Posts: 27
Joined: Thu Sep 22, 2011 11:11 am

Re: bp shinners’ semi-weekly office hours

Post by ruhl88 » Tue Nov 20, 2012 11:23 am

The substitution rules on the more recent games sections are killing me. Any way to effectively tackle these types? Thanks!

meandme

New
Posts: 74
Joined: Wed Feb 15, 2012 9:36 pm

Re: bp shinners’ semi-weekly office hours

Post by meandme » Tue Nov 20, 2012 2:53 pm

bp shinners wrote:
meandme wrote:Hey Matt
How would I diagram this?
V cannot be in unless both H and M are in.

I thought I could just read the "unless" as "if not". This what I got

~h and ~m --> ~v
CP V --> H or M

The "both" is throwing me off.

Thanks
Yep, you can just change 'unless' to 'if not'. However, you only negated H and M when you did that - you also have to negate the 'and'.

So "If not both H and M" is the same as saying, "If I don't have H or M." That diagram as:
~H or ~M -> ~V
V-> H and M

And that's what the original is saying - if I have V, then I have both H and M.
Which can be broken down into
V -> H
V -> M

Oh, is "nor" the negated "and"

If W is in, neither H nor X is in.
W -> ~H
W -> ~M
I know you had told me I could split the statement when the "and" is in the outcome.

Thanks

Register now!

Resources to assist law school applicants, students & graduates.

It's still FREE!


User avatar
bp shinners

Gold
Posts: 3086
Joined: Wed Mar 16, 2011 7:05 pm

Re: bp shinners’ semi-weekly office hours

Post by bp shinners » Tue Nov 20, 2012 8:33 pm

ruhl88 wrote:The substitution rules on the more recent games sections are killing me. Any way to effectively tackle these types? Thanks!
There are three types of questions that mess with the rules:

1) A completely new rule is added (see Jabrohn getting his car washed)
For these, start from square one. Copy over your old rules, write down the new one, and make a new setup (with new deductions).

2) A rule is removed
Same method as 1

3) "Which one of the following, if substituted for rule X, would have the exact same effect on the game?"
This is the tough one, and it's the one that most people have trouble with.

A little background. Normally, they ask you to replace 2-part rules (something like, "H can't be first, and H is before K."). If it's a one-part rule, then skip the second step.

Here's my process:
A correct answer choice will have 2 or 3 things true of it (depending on a 2- or 1-part rule)
1) It will tell you the first part of the rule
2) It will tell you the second part of the rule (if one exists)
3) It will tell you nothing else.

Analyzing the answer choices over metrics 1 and 2 are significantly easier than analyzing it over metric 3. So start with those. Eliminate any answer choice that doesn't guarantee you that the first/second part of the original rule is followed. This will often leave you with 1 answer, but it's just as likely to still have 2-3. If that's the case, you're looking to eliminate answers that tell you too much. For instance, if a rule tells me F must be first, then I know H can't be as well (going form my example above), but now I also know that F must be first - a new piece of info. This isn't the answer you're looking for.

When you find something that satisfies all three criteria, you're golden.

A final note - these questions will invariably take more time. It's just how it goes. Luckily, the LSAT is designed around having these questions included, so the other questions are designed to take a little less time because of them. So don't worry too much about the timing on these - they will take awhile.

User avatar
bp shinners

Gold
Posts: 3086
Joined: Wed Mar 16, 2011 7:05 pm

Re: bp shinners’ semi-weekly office hours

Post by bp shinners » Tue Nov 20, 2012 8:35 pm

meandme wrote: Which can be broken down into
V -> H
V -> M
Exactly.
Oh, is "nor" the negated "and"

If W is in, neither H nor X is in.
W -> ~H
W -> ~M
I know you had told me I could split the statement when the "and" is in the outcome.

Thanks
I wouldn't say 'nor' is the negated 'and'; I would say 'nor' is the same as 'and', but used in phrases where you're saying not A AND not B (neither A nor B). If you negate 'and', it becomes 'or'.

Outside of that (semantic) argument, though, you're exactly right with your example.

ruhl88

New
Posts: 27
Joined: Thu Sep 22, 2011 11:11 am

Re: bp shinners’ semi-weekly office hours

Post by ruhl88 » Sun Nov 25, 2012 8:20 pm

bp shinners wrote:
ruhl88 wrote:The substitution rules on the more recent games sections are killing me. Any way to effectively tackle these types? Thanks!
There are three types of questions that mess with the rules:

1) A completely new rule is added (see Jabrohn getting his car washed)
For these, start from square one. Copy over your old rules, write down the new one, and make a new setup (with new deductions).

2) A rule is removed
Same method as 1

3) "Which one of the following, if substituted for rule X, would have the exact same effect on the game?"
This is the tough one, and it's the one that most people have trouble with.

A little background. Normally, they ask you to replace 2-part rules (something like, "H can't be first, and H is before K."). If it's a one-part rule, then skip the second step.

Here's my process:
A correct answer choice will have 2 or 3 things true of it (depending on a 2- or 1-part rule)
1) It will tell you the first part of the rule
2) It will tell you the second part of the rule (if one exists)
3) It will tell you nothing else.

Analyzing the answer choices over metrics 1 and 2 are significantly easier than analyzing it over metric 3. So start with those. Eliminate any answer choice that doesn't guarantee you that the first/second part of the original rule is followed. This will often leave you with 1 answer, but it's just as likely to still have 2-3. If that's the case, you're looking to eliminate answers that tell you too much. For instance, if a rule tells me F must be first, then I know H can't be as well (going form my example above), but now I also know that F must be first - a new piece of info. This isn't the answer you're looking for.

When you find something that satisfies all three criteria, you're golden.

A final note - these questions will invariably take more time. It's just how it goes. Luckily, the LSAT is designed around having these questions included, so the other questions are designed to take a little less time because of them. So don't worry too much about the timing on these - they will take awhile.
I'm still having trouble with these question types. Can you run through a game question for me so I can see this process from your perspective? How about PT61 G2 Q11? Again, thanks so much for your help!

User avatar
bp shinners

Gold
Posts: 3086
Joined: Wed Mar 16, 2011 7:05 pm

Re: bp shinners’ semi-weekly office hours

Post by bp shinners » Mon Nov 26, 2012 12:53 pm

ruhl88 wrote: I'm still having trouble with these question types. Can you run through a game question for me so I can see this process from your perspective? How about PT61 G2 Q11? Again, thanks so much for your help!
No problem!

So the rule I'm trying to replace says that the necklace and jar are older than the tureen. That's a two part rule, so my new rule is going to tell me:
1) The necklace must be older than the tureen
2) The jar must be older than the tureen
3) Nothing else.

A - This rule creates no relationship between either the necklace or the jar and the tureen. It fails on 1 and 2, so I don't have to consider it any more.

B - This rule tells me the necklace is older than the tureen; it satisfies 1. However, I have no relationship between the jar and tureen, so it's out for not satisfying 2.

C - This rule gives me a relationship between the necklace, jar, and tureen. However, it tells me they're either both older OR both younger than the tureen.

Now, if there's another rule that would prevent the second scenario from happening, then this would be the answer. However, there's nothing preventing both J and N from being younger than the T, so this one's out as well (it doesn't tell me 1 or 2, since they could be older, but they could also be younger; I need the rule to tell me they MUST BE older).

D - Everything but the H and P must be older than the T. Well, that includes N and J. Bingo on 1 and 2. I'm going to punt on analyzing across 3 for a minute.

E - Nothing in this rule relates the J and the T. So it's going to fail on 2. (It also fails on 1, but as soon as I see it fails on 2, I stop looking.)

D is the only one that gives me both parts of the rule, so it must be the right answer (even though we didn't see if it gave us too much info). Going back to check that it doesn't give us too much info, we see it doesn't.

If all the artifacts except H and P must be older than the tureen, that tells me nothing about H and P (since they could be older or younger; they just don't have to be older). So no new info there. The list of objects that must be older than the T are J, N (my two that I was looking for), and F. Uh-oh; I wasn't looking to say F must be older than the T! However, I know F must be older than J from another rule, so I already knew that F must be older than T (since it's older than J and J's older than T). D meets all 3 criteria for a correct answer, so it's what I'm going with.

Get unlimited access to all forums and topics

Register now!

I'm pretty sure I told you it's FREE...


User avatar
BlaqBella

Silver
Posts: 868
Joined: Fri Jan 28, 2011 9:41 am

Re: blueprint shinners’ semi-weekly office hours

Post by BlaqBella » Wed Nov 28, 2012 11:32 am

Hey Matt!! My below question is two-fold.

I've identified two types of principle questions and want to know if I am correct in this approach:

1. Those that apply a principle (you are given a principle and you need to apply it to a scenario). I call this applied principle; and

2. Those where you have a argument and you need to support it with some general application (or principle, if you will). I call this principle support.


And my second question: how does principle support questions differ from justify/paradox questions? Or for that matter a strengthen question?? Are these not overlapping in question type identification?

ruhl88

New
Posts: 27
Joined: Thu Sep 22, 2011 11:11 am

Re: bp shinners’ semi-weekly office hours

Post by ruhl88 » Wed Nov 28, 2012 4:12 pm

bp shinners wrote:
ruhl88 wrote: I'm still having trouble with these question types. Can you run through a game question for me so I can see this process from your perspective? How about PT61 G2 Q11? Again, thanks so much for your help!
No problem!

So the rule I'm trying to replace says that the necklace and jar are older than the tureen. That's a two part rule, so my new rule is going to tell me:
1) The necklace must be older than the tureen
2) The jar must be older than the tureen
3) Nothing else.

A - This rule creates no relationship between either the necklace or the jar and the tureen. It fails on 1 and 2, so I don't have to consider it any more.

B - This rule tells me the necklace is older than the tureen; it satisfies 1. However, I have no relationship between the jar and tureen, so it's out for not satisfying 2.

C - This rule gives me a relationship between the necklace, jar, and tureen. However, it tells me they're either both older OR both younger than the tureen.

Now, if there's another rule that would prevent the second scenario from happening, then this would be the answer. However, there's nothing preventing both J and N from being younger than the T, so this one's out as well (it doesn't tell me 1 or 2, since they could be older, but they could also be younger; I need the rule to tell me they MUST BE older).

D - Everything but the H and P must be older than the T. Well, that includes N and J. Bingo on 1 and 2. I'm going to punt on analyzing across 3 for a minute.

E - Nothing in this rule relates the J and the T. So it's going to fail on 2. (It also fails on 1, but as soon as I see it fails on 2, I stop looking.)

D is the only one that gives me both parts of the rule, so it must be the right answer (even though we didn't see if it gave us too much info). Going back to check that it doesn't give us too much info, we see it doesn't.

If all the artifacts except H and P must be older than the tureen, that tells me nothing about H and P (since they could be older or younger; they just don't have to be older). So no new info there. The list of objects that must be older than the T are J, N (my two that I was looking for), and F. Uh-oh; I wasn't looking to say F must be older than the T! However, I know F must be older than J from another rule, so I already knew that F must be older than T (since it's older than J and J's older than T). D meets all 3 criteria for a correct answer, so it's what I'm going with.
I am no longer afraid of these questions. You are the best! Thank you!

ruhl88

New
Posts: 27
Joined: Thu Sep 22, 2011 11:11 am

Re: blueprint shinners’ semi-weekly office hours

Post by ruhl88 » Wed Nov 28, 2012 4:16 pm

Shinners,

PT 62 S4 Q18 - this one really disrupted my flow on the timed exam. Can you explain this? And, on a more general LR level, how do I mitigate disruptions like this? I usually know the answer to most of the LR questions but I tend to get tripped up by arguments that appear complex and are verbose. TIA!

meandme

New
Posts: 74
Joined: Wed Feb 15, 2012 9:36 pm

Re: blueprint shinners’ semi-weekly office hours

Post by meandme » Wed Nov 28, 2012 10:56 pm

Hey Matt
On pt10 LG2 why do we put reviewers to movies and not the other way around?

Thanks

Communicate now with those who not only know what a legal education is, but can offer you worthy advice and commentary as you complete the three most educational, yet challenging years of your law related post graduate life.

Register now, it's still FREE!


User avatar
bp shinners

Gold
Posts: 3086
Joined: Wed Mar 16, 2011 7:05 pm

Re: blueprint shinners’ semi-weekly office hours

Post by bp shinners » Thu Nov 29, 2012 9:57 am

BlaqBella wrote:Hey Matt!! My below question is two-fold.

I've identified two types of principle questions and want to know if I am correct in this approach:

1. Those that apply a principle (you are given a principle and you need to apply it to a scenario). I call this applied principle; and

2. Those where you have a argument and you need to support it with some general application (or principle, if you will). I call this principle support.


And my second question: how does principle support questions differ from justify/paradox questions? Or for that matter a strengthen question?? Are these not overlapping in question type identification?
Alright, so at Blueprint, we recognize 3 types of principle questions (two of which you've identified):

1) Soft Must Be True Principle (which you call applied principle)
"Which one of the following most closely conforms to the principle stated above?"

Here, we get the principle (aka general rule) in the stimulus, and a bunch of situations in the answer choices. The principle is likely something that you can diagram (and, in fact, these tend to be the diagrammable soft must be true questions, as most of them aren't diagrammable).

Method? First, diagram the principle. There are usually two principles, actually, and they usually contain value judgments that, if not opposed, are related (pleasant/oppressive days; morally right/wrong; good/bad idea). Whatever the related value judgments are, get them to your necessary side (if they aren't given to you that way, take the contrapositive). Then, analyze the answer choices using those rules.

You can generally quickly eliminate answer choices that can't possibly be right based on what you know. So, for instance, if I have Sunny + Cool Breeze + Under 85 degrees = Pleasant Day, and that's my only principle, an answer choice that tells me something isn't a Pleasant Day can't possibly be right since I don't know anything that's sufficient to tell me a day isn't Pleasant. If you have two principles, you have to see if the conclusion can be proven by either one, but hopefully you get the point.

These are the opposite of....

2) Strengthen Principle (which you call principle support)
"Which one of the following principles most justifies the actions of Sharon?"

Here, we get the situation in the stimulus and are asked to find an AC that justifies the conclusion (the opposite of above, where we get the situation in the AC and principle in the stimulus). These seem tricky at first, but once you get the hang of them, they actually become quite easy.

So in these questions, I'm going to have a situation with some type of flaw in it. You can find the flaw if you want, but the basics of these questions doesn't require it. Instead, I'm looking for a general rule/principle that simply states, "If (premises), then (conclusion)." It sounds dumb. Really dumb. The first time I predict an answer like this in my class, students laugh and think it can't be that easy. It is.

You're simply looking for an AC that, in general terms, tells you that your premises justify/guarantee your conclusion.

The same caveat applies here - if my conclusion is that the government should spend the money, I need that in my AC (not that they shouldn't, and not that they should be allowed to - I need to justify that they should). There's usually 2 ACs that are picked for this type of question - the right one, and the sucker one. The right one justifies the actual conclusion (the government should spend money on space travel), and one that justifies the close-but-no-cigar conclusion that doesn't appear in the stimulus (the government shouldn't be prevented from spending money on space travel).

These are the same as justify questions.

3) Parallel Principle Questions
"Which one of the following situations most closely conforms to the principle underlying the situation above?"

Here, I get a situation in BOTH the AC and stimulus. I need to come up with a general rule that underlies both the stimulus and AC. You didn't ask about this one, but I'm throwing it in for free.

What you want to do for these is, essentially, describe the rule that would justify the conclusion in the stimulus, but strip it of any subject matter. Then, do the same with the ACs until you explain it in the same way. Like all parallel questions, the burden for a correct answer is very high, so don't fudge it.

Follow up Qs:
We've addressed justify questions.

Paradox questions are completely different from these. We call them resolve/explain questions. They're a weird beast (though they are my favorite question types on the exam) because the ACs are a little 'looser' in logic than the rest of the test. There's a little more leeway for you to make some jumps - it's not MBT levels of certainty for these. The way I always phrase these in my head is, "On the one hand (x) is true; on the other, (y) is true." I need to find an answer that says X is still true, Y is still true, and now I understand how they can coexist.

User avatar
bp shinners

Gold
Posts: 3086
Joined: Wed Mar 16, 2011 7:05 pm

Re: blueprint shinners’ semi-weekly office hours

Post by bp shinners » Thu Nov 29, 2012 10:10 am

ruhl88 wrote:Shinners,

PT 62 S4 Q18 - this one really disrupted my flow on the timed exam. Can you explain this? And, on a more general LR level, how do I mitigate disruptions like this? I usually know the answer to most of the LR questions but I tend to get tripped up by arguments that appear complex and are verbose. TIA!
So first things first - how to mitigate a question like this. Two pieces of advice.

1) If you run into something like this that's just killing your flow, skip it. There's nothing wrong with coming back to it after you've built up a little bit of confidence with a few more questions. Plus, then you'll know how long you have to spend on it.

2) Go back to the basics. Break it down one sentence at a time. There are no complex ideas here, just complex wording/phrasing. When you break these verbose/complex questions down, they're usually composed of some very basic propositions.

So first thing I notice: I start with a qualified conclusion, and it's conditional. We qualify it by saying that, "If aliens exist...". Well, fine, I'll give him that. Which is good, because now I don't have to worry about it - since he qualified his conclusion, I know that he's not assuming their existence - everything that he argues is based on assuming they might not exist, but, for the sake of argument, let's say they do. So since he qualified his conclusion, I can pretty much forget about the debate over the existence of aliens, because if they don't exist, then they author's conclusion isn't wrong; it just doesn't apply at all.

So my conclusion is: "If ET isn't as smart as people, we won't know he exists." Why?
1) We don't have some crazy Star Trek-style warp drive (to send ships out of the solar system)
2) If ET wants to talk with us, he has to be at least as intelligent as we are

So, essentially, we can't visit him, and if he's not as smart as us, he can't talk to us (guess he's not phoning home after all).

When I have a sufficient assumption question, I look for new ideas in the conclusion. Here, it's knowing about ETs existence - I only talk about methods of detecting ET; I don't talk about knowing whether he exists or not. However, I do have the other idea in the conclusion - him not being as smart as us.

So trying to connect my premises to my conclusion, I see overlap between ET not being as smart as us, so that's where I start. My conclusion states it, and my second premise states it. And it tells me that, if that's true, he can't talk to us.

I add to that the fact we can't visit him. So no talking, no visiting, no touching (that one's for the Arrested Development fans out there).

Is not being able to visit and not being able to talk enough to let me know we won't know they exist? I don't think so. Maybe there are other ways for us to figure out they exist (a super space telescope, perhaps). So this is an exclusivity fallacy - I rule out 2 ways of discovering the existence of ET, and from that conclude that it's a lost cause. Well, there could very well be other ways of finding out he exists (finding a crashed spaceship on Mars, for instance).

If I want my conclusion to be valid, I need to establish that these are, in fact, the only two ways to find out if ET exists. And D gives me that - if they can't talk with us (i.e. they aren't as smart as us), then sending a ship to them is the only way to find out if they exist (important - it includes the new term that showed up in my conclusion; that's necessary for a sufficient assumption question).

User avatar
bp shinners

Gold
Posts: 3086
Joined: Wed Mar 16, 2011 7:05 pm

Re: blueprint shinners’ semi-weekly office hours

Post by bp shinners » Thu Nov 29, 2012 10:12 am

meandme wrote:Hey Matt
On pt10 LG2 why do we put reviewers to movies and not the other way around?

Thanks
"Each reviewer reviews exactly one movie, and each movie is reviewed by at least one of the six reviewers."

That lets me know that I place each reviewer once into movies. When it's set like that (and the other variable set is unstable), I use the unstable variable set for my groups.

On top of that, my rules all create grouping relationships between my reviewers (Hilda and Frank).

User avatar
bp shinners

Gold
Posts: 3086
Joined: Wed Mar 16, 2011 7:05 pm

Re: blueprint shinners’ semi-weekly office hours

Post by bp shinners » Tue Dec 04, 2012 5:03 pm

February's closer than you think. Let's get the studying started!

Seriously? What are you waiting for?

Now there's a charge.
Just kidding ... it's still FREE!


Post Reply

Return to “Free Help and Advice from Professionals”